Home
Current Affairs January 2024

What is the correct answer?

4

Which statement is true about the female urethra?

A. harder to catheterize when compared to the male urethra

B. 4 inch long

C. opens into the vestibule below the clitoris

D. gets wider at the post-menopausal age

Correct Answer :

C. opens into the vestibule below the clitoris


the female urethra is 4 cm long, easier to catheterize, and gets narrow atrophic post menopausally due to estrogen deprivation.

Related Questions

What is the correct answer?

4

What is the nerve supply of the adrenal gland?

A. sympathetic to the medulla

B. parasympathetic to the medulla

C. sympathetic to the cortex

D. parasympathetic to the cortex

What is the correct answer?

4

To which region does pain, from the left mid ureter, refer to?

A. the loin region

B. the inguinal region

C. the umbilical region

D. the penis or clitoris

What is the correct answer?

4

Bacterial resistance to antibiotics may occur because of:

A. inherited chromosomal-mediated resistance

B. acquired chromosomal-mediated resistance

C. extrachromosomal-mediated resistance

D. all of the above

What is the correct answer?

4

Which organ contributes most to the seminal fluid volume?

A. testes

B. prostate

C. seminal vesicles

D. bulbourethral glands

What is the correct answer?

4

Which of the following is false regarding zona glomeruloza?

A. the most superficial layer of the adrenal cortex

B. responses to increased potassium levels, renin or decreased renal blood flow

C. it causes pheochromocytoma

D. secretes aldosterone

What is the correct answer?

4

Which part of the prostate is traversed by the ejaculatory duct:

A. central zone

B. peripheral zone

C. transitional zone

D. fibromuscular stroma

What is the correct answer?

4

In which body tissue(s) are creatine and phosphocreatine converted to creatinine?

A. bone marrow

B. skeletal muscles

C. brown fat

D. liver

What is the correct answer?

4

Where is Santorini plexus located?

A. at either side of the prostate

B. in the pubo-prostatic space

C. anterior to the seminal vesicles

D. posterior to the vaso-epididymal junction

What is the correct answer?

4

Hematuria of glomerular origin is associated with:

A. significant proteinuria, dysmorphic RBCs, RBC casts

B. glycosuria, eumorphic RBCs, WBC casts

C. hypercalciuria, eumorphic RBCs, granular casts

D. proteinuria, dysmorphic RBCs, hyaline casts

What is the correct answer?

4

Which statement is false concerning the gubernaculua?

A. degenerate after birth

B. called the caudal genital ligaments

C. help guide the testes down through the inguinal canals

D. represent undifferentiated mesenchyme

What is the correct answer?

4

Which structure is a part of the renal hilum?

A. renal pelvis

B. branches of the renal artery

C. tributaries of renal vein

D. all of the above

What is the correct answer?

4

Which of the renal artery occlusive conditions, commonly, do NOT affect renal function?

A. intimal fibroplasia

B. medial fibroplasia

C. medial hyperplasia

D. serosal hyperplasia

What is the correct answer?

4

In which organelle in Leydig cell does testosterone synthesis take place?

A. ribosome

B. mitochondria

C. Golgi apparatus

D. endoplasmic reticulum

What is the correct answer?

4

Which statement is false concerning capacitation of spermatozoa?

A. occurs after acrosome reaction

B. entails removal of a glycoprotein layer

C. it is a biochemical event of sperm maturation

D. changes occur in the female genital tract

What is the correct answer?

4

Which agent is most likely to cause hemorrhagic cystitis?

A. cisplatin

B. ifosfamide

C. phosphomycin

D. amphotericin B

What is the correct answer?

4

Which of the following is (are) direct branch(es) of the abdominal aorta?

A. median sacral artery

B. inferior phrenic arteries

C. gonadal arteries

D. all of the above

What is the correct answer?

4

Which lab results indicate poor prognosis of male infertility?

A. azoospermia; normal FSH; increase LH

B. azoospermia; decrease FSH; normal LH

C. oligospermia; increase FSH; normal LH

D. oligospermia; normal FSH; decrease LH

What is the correct answer?

4

Which of the following does asymptomatic hematuria workup include?

A. cystoscopy

B. CT urography

C. urine cytology

D. all of the above

What is the correct answer?

4

What is the approximated bladder capacity of an 8 yrs. girl?

A. 150 ml

B. 200 ml

C. 250 ml

D. 300 ml

What is the correct answer?

4

What causes Cushing`s disease is:

A. bilateral adrenal hyperplasia

B. pituitary over secretion of ACTH

C. exogenous administration of glucocorticoids

D. immune-mediated disease

What is the correct answer?

4

Which of the following is NOT a part of the urogenital diaphragm structures?

A. perineal membrane

B. deep transverse perineal muscle

C. urethral sphincter

D. Camper`s fascia

What is the correct answer?

4

What is false about indinavir sulfate stones?

A. intratubular crystal formation might occur

B. CT cannot reliably confirm the presence of indinavir calculi

C. stone formation is demonstrated in 80% of patients taking the medication

D. is a protease inhibitor with poor solubility and significant urinary excretion

What is the correct answer?

4

Which of the following is NOT a component of the normal semen?

A. zinc

B. fructose

C. prostate-specific antigen

D. alkaline phosphatase

What is the correct answer?

4

The mature genitourinary system develops from all of the following embryonic sources, EXCEPT:

A. intermediate mesoderm

B. mesothelium of celomic cavity

C. endoderm of the urogenital sinus

D. somatic ectoderm

What is the correct answer?

4

How many renal papillae are there in a typical kidney?

A. 4 to 6

B. 7 to 9

C. 10 to 12

D. 13 to 14

What is the correct answer?

4

What is the best method to evaluate calcium levels in the urine?

A. from early morning urine sample

B. comparing urine and serum calcium at a given time

C. performing 24 urine collection

D. from a mid-stream urine sample

What is the correct answer?

4

Which disease is described best by the following manifestations: grand mal epilepsy, mental retardation, hypertension, angiomyolipoma, adenoma sebaceum, bilateral enlarged kidneys, and normal renal function?

A. autosomal dominant polycystic kidney disease

B. von Hippel-Lindau disease

C. tuberous sclerosis

D. Sturge-Webber syndrome

What is the correct answer?

4

In which condition can NOT creatinine level be 1200 mg/dL?

A. an aspirate of pelvic urinoma

B. in untreated end-stage renal failure disease

C. a sample from a wound drain after pyeloplasty

D. a sample from suprapubic catheter

What is the correct answer?

4

Which class of the following diuretics could cause erectile dysfunction?

A. loop diuretics

B. thiazide diuretics

C. potassium sparing diuretics

D. osmotic diuretics

What is the correct answer?

4

Where do Cowper`s gland ducts drain into?

A. ejaculatory ducts

B. prostatic urethra

C. membranous urethra

D. bulbous urethra